« first day (1684 days earlier)      last day (3355 days later) » 

10:00 AM
@infinitesimal I am aware of the distinction between "a-thing-in-itself" and the symbol we use to represent it, certainly $9$ and $IX$ refer to some quality common to both descriptions
 
@TheArtist The Archangel of integrals came to me one night and handed me the Book of Knowledge with all Integrals, series and limits a man can think of. That was the start ... (I'm just kidding ...:-))
 
Neurobiologists seem to be of the opinion lately that there is a certain region of the brain with a highly-localized and specific function for distinguishing quantity
@Chris'ssis This Archangel-he, or she? What did they look like?
2
 
@Chris'ssis the real story ? :)
 
@TheArtist Just kidding as I wrote above. :-)
 
@Chris'ssis I know :) I mean tell the real story....everyone wants to hear :)
 
10:06 AM
Things they don't tell you in Calculus: those (open/closed) "intervals" are the skeleton they hang the meat on. Nothing makes sense without limits.
I'm surprised people don't openly revolt and demand to know: "What the heck ARE these interval-thingies?"
 
@TheArtist I began with a textbook one of my cousines didn't use anymore since she gave up the high school, it was a book ready for thrash bin. I took it, studied it and then I began to like more and more the stuff in it, especially integrals, series and limits. The story is far more complex than that, of course.
 
Integral of a constant function over a closed loop is $0$ ? $\oint_{\partial U }dx t = 0 $ ?
 
\oint
 
@DavidWheeler thanks
of course $\oint_{\partial U} dx =0$
 
@Chris'ssis wow
@Chris'ssis that's a wonderful start (respect!)
 
10:11 AM
@TheArtist The book was prepared to be thrown in a thrash bin. It sounds crazy, I know, but it is really so.
 
@Chris'ssis What was the first thing that really gave you trouble?
 
@DavidWheeler I remember that was weird to me to find out that I add up infinitely many terms and I cannot get a value more than $0$.
 
@Chris'ssis wow :)
 
@DavidWheeler While I was studying this limit $$\lim_{n\to\infty} \sum_{k=1}^{n} \frac{1}{n+k}$$
That was at the very beginning, one of the things I was pondering over. The more I was studying, the more fascination I was founding in this stuff.
 
Yeah, infinity does strange things to our reasoning process.
When you start investigating series, you're getting "deep" into the structure of real (and later complex) numbers.
 
10:18 AM
@DavidWheeler The most impressive of all was the Basel series I found at one of the pages. There was no proof nowhere, so I was thinking I might find one at that time.
However, I couldn't imagine how one can possibly get that very strage $\pi^2/6$. It looked to me like some sort of magical thing more than something scientifical.
That series marked me profoundly. I remember that I was skimming book after book in a library hoping to find that series with a proof and found anything for a while. At that time I didn't have internet, so less possibilities for information.
I didn't have rest until I found that magical explanation (one of them, involving Fourier series) behind the very beautiful result.
 
In high-school, my calculus teacher used Fourier series for $x^n$ to come up with values for $\pi^n$.
He remarked-but I can't find the pattern for the coefficient of the power.
 
Now there are tons of proofs to the Basel problem. I also sent one to AMM to be published. I'm waiting to see if they are going to publish it.
 
So I worked on it, for a long, long time-and rediscovered Bernoulli and Euler numbers. I was like: shoot, someone's already done this.
@Chris'ssis Good luck on your submission!
 
@DavidWheeler Thanks. :-)
For a long while I was asking questions and I was the only one to answer them. That's the disadvantage of being self-educated, there is no one to give you a hand when needed.
 
Since not every function has a primitive, every integral actually found is a major victory.
It's like charting an unknown country-we might know the law of gravity, but that doesn't help with navigation.
And you never know when a certain class of functions might prove to be a desirable basis of a function space for applications we don't know about yet.
 
10:32 AM
I said many things on MSE, this is true though, although it sounds ridiculuosly crazy: starting from a book that was prepared to be thrown at trash bin. :-)
That's to answer @TheArtist's question.
 
one person's trash is another person's treasure. :)
5
 
@Chris'ssis whats the solution for $$\lim_{ n \to \infty} \sum_{k=1}^{k=n} \frac{1}{ n+k}$$
 
Yeah. :-)))
@Theorem Can't you do it?
 
@Chris'ssis :)
 
@Chris'ssis at first look my thought was that it was similar to harmonic series
 
10:35 AM
@DavidWheeler totally agree :)
 
@Theorem It can be done in more ways, but yes, using harmonic numbers is a very simple way to do it.
 
@Chris'ssis it's quite late here :) I will catch u later. Thanks for sharing your wonderful story :) bye
 
@TheArtist bye ;)
 
@Chris'ssis :)
 
@Chris'ssis would like to know how to solve it .
 
10:36 AM
@Theorem how can you write in terms of harmonic numbers $$\sum_{k=1}^{k=n} \frac{1}{ n+k}$$?
 
I can see it's easily between 1/2 and 1
 
Note that $$\sum_{k=1}^{k=n} \frac{1}{ n+k}=H_{2n}-H_n$$
 
Hello! I have a permutation group that is central symmetric after the group operation has been applied a certain number of times. How could I go about proving this?
E.g. (A,B,1,2) becomes (2,1,B,A)
 
$$\sum_{k=1}^{k=n} \frac{1}{ n+k}=(H_{2n}-\log(2n))-(H_n-\log(n))+\log(2n)-\log(n)$$
@Theorem take the limit and you're done. Q.E.D.
 
So $\gamma - \gamma + \log(2) = \log(2)$
 
10:44 AM
@Chris'ssis That was a cool one :D
 
That's about 0.6, so my crude estimates were OK
 
@Theorem :D
 
It's interesting, because at first glance it "feels" like it should be unbounded, but it's not.
That extra $n$ in the denominator is enough to keep it starting off slow enough to converge.
 
$\sum_{n=2} \frac{1}{n ln ln n}$ , what do you think about the convergence speed of these two ?
 
Math joke: what does an analyst say when he's drowning?
$\log\log\log\log....$
 
10:49 AM
I am converging
 
@DavidW I thought I had this question, but apparently it goes wrong almost immediately: meta.math.stackexchange.com/a/11872/142198
Every eigenvalue of $J$ need not be $1$ here?
Is this even salvageable?
 
Hi my dear friends.
 
@ABeautifulMind Hi :-)
 
@ABeautifulMind Hi, I am dying from assignment overdose
 
@Chris'ssis I went out for a walk today.
 
10:52 AM
@ABeautifulMind That's great!
 
@Committingtoachallenge I think you should not overwork yourself. Your challenge is too ambitious.
 
@ABeautifulMind This is just for the class
Abstract Algebra II (MATH3303)
 
@Committingtoachallenge No, not every eigenvalue need be one. Think of $n = 2$ and $-I$.
 
@Committingtoachallenge I suggest you just focus on your class work and forget about the extra books. Also there is no point following my list of books, hehe.
 
@ABeautifulMind This is just class work haha
@DavidWheeler Argh I am so broken
 
10:53 AM
I was already affected by mental problems in undergrad, so I only got a second upper in the end.
 
@DavidWheeler Maybe my proof doesn't need that actually, it will still be diagonal I think
 
Actually I missed first class by one grade in one course, lol.
 
What you do know, is that the eigenvalues must be $n$-th roots of unity.
 
I needed 4.5 out of 5 to get first class, and I got 4.49, lol.
 
if $A$ is $n \times n$ what algebraic multiplicity can an eigenvalue have?
 
10:56 AM
$n$
Are you saying I have $n$ distinct roots?
Which would give me no $0$'s on the super diagonal
Hence all $1\times 1$ blocks
Which means it is diagonal?
Sorry all $0$'s no $1's$
Yeah I think I got it thanks!
Alternatively I got all of that wrong and I am just really tired LOL
 
The Jordan Normal Form is difficult, because not all matrices "behave well"
 
I do have $n$ distinct roots right?
in $\Bbb C$
 
Can you prove this, by exhibiting them?
 
I have n distinct teeth, lol.
I have 26 teeth because I extracted 2.
Also, I have no wisdom tooth.
Wow, that killed the chat.
 
@DavidWheeler, do you know offhand the cardinality of the Hilbert Cube? In particular, is $\displaystyle |\mathbb{R}| = \left| \prod_{k=1}^\infty [0, 1] \right|$ ?
 
11:06 AM
$\def\b{\begin{bmatrix}}\def\e{\end{bmatrix}}$
@DavidW That student is still disagreeing with me, but I don't know how to prove it, am I wrong?

Is $\b 0&1&0\\0&0&0\\0&0&0\e$ perfectly fine jordan normal form with all eigenvalue $0$?
With a 2x2 jordan block and a 1x1 jordan block
Or does the same eigenvalue force it to become a 3x3 jordan block $\b0&1&0\\0&0&1\\0&0&0\e$
@kaj do you know the answer to above?
 
Oh lord @Committingtoachallenge, my linear algebra is rusty :/
 
Is it made of iron?
 
My XXX is made of iron.
 
@Kaj that's alright, I am just dying lol. I have spent 42hours plus including today on this damn assignment, and I am in week 2
Each class is meant to take 12 hours a week, and this is just ridiculous
I have spent more time on this assignment than on my other 4 classes combined
 
11:10 AM
What class is this?
 
@ABeautifulMind Abstract algebra II
 
They both have the same characteristic polynomial: $x^3$. It's clear the second one has minimal polynomial $x^3$ as well. What is the minimal polynomial of the first one?
 
Oh no its x is it?
 
11:12 AM
no. it's not the 0-matrix
 
Apparently I am still confused on what min and characteristic polynomials are
which is crap
 
You ran across the campus crap that day, lol.
 
I think I must be wrong then
The second has min poly x^3
But the first is just wrong, since it must have x^2
 
Yes, that's obvious from its nilpotency
 
Second has nilpotency 3
First has nil 2
So I screwed up
So my giant proof is wrong
So I wasted hours
 
11:15 AM
Now, suppose the first matrix was what we had "before" we converted to Jordan form.
 
You can see that $\{(1,0,0),(0,0,1)\}$ form a basis for $E_0$, the eigenspace of 0.
 
Hello @KajHansen @DavidWheeler :) Are you familiar with differential equations?
 
Is that right @DavidW
 
@evinda Not really. I can sometimes do linear ODE, but PDE are harder, haven't read much on them.
 
11:18 AM
A ok @DavidWheeler No problem :)
 
@Committingtoachallenge Is what right?
 
Oh wait nvm I see
That is right
 
To get a basis, we need to add a vector in $\text{ker }(0I - A)^2$
That's just 1 extra generalized eigenvector, so the Jordan form should only have one 1 on the super-diagonal.
 
I don't understand
A basis for what sorry?
 
by contrast, the second matrix has only one eigenvector (1,0,0)
 
11:23 AM
true
 
to get a basis, we need to get a vector from $\text{ker }(0I - B)^2$, AND one from $\text{ker }(0I - B)^3$
 
Hello!! I am looking at the problem $$u_{tt}(x, t)-u_{xt}(x, t)=0 \\ u(x, 0)=f(x) \\ u_t(x, 0)=g(x)$$

Does it stand that $$u_{xt}=u_{tx}$$ ??
 
So is it no coincidence with those powers? I have not seen this before
2,3 correspond to the position in the eigenvector that is null
 
There's no way that the two situations are in any way "similar"-$E_0$ has dimension 2 in the first scenario, and dimension 1 in the second.
 
But that doesn't answer if the first is a valid jordan block(even though that definitely fixed my understanding of somethings)
 
11:26 AM
remember, using a "change of basis" matrix isn't going to change the dimension of $E_0$, since those are bijective maps.
 
@DanielFischer @robjohn @anon @quid

I am looking at the problem $$u_{tt}(x, t)-u_{xt}(x, t)=0 \\ u(x, 0)=f(x) \\ u_t(x, 0)=g(x)$$

Does it stand that $$u_{xt}=u_{tx}$$ ??
 
I mean combination of jordan blocks
 
@Committingtoachallenge What I'm saying is, there is NO WAY, if we started with the first matrix, we would wind up with the 2nd under ANY similarity transform.
 
Okay I must not understand something, I just wasn't sure if we were allowed to take $C={J_1}_{2\times 2} \bigoplus {J_2}_{1\times 1}$ for $$ C =
\left(
\begin{array}{cc|c}
0 & 1 & 0 \\
0 & 0 & 0 \\ \hline
0 & 0 & 0
\end{array}
\right).
$$
 
The first matrix satisfies $A^2 = 0$. If $PAP^{-1} = B$, then we would have: $B^2 = (PAP^{-1})(PAP^{-1}) = PA^2P^{-1} = P0P^{-1} = 0$.
But $B^2 \neq 0$, it's of nilpotency 3
 
11:30 AM
Where both $J_1$ and $J_2$ have eigenvalue only $0$
 
@Committingtoachallenge And I'm trying to convince you we CAN, because if we were forced to have the second matrix as the first matrix's Jordan form, we get a contradiction.
 
Oh man how embarrassing for me
Oh man it is so obvious now
 
3x3 matrices of nilpotency 2 exist, you just exhibited one.
 
(that you were trying to show that you can...)
 
@MaryStar yes I would say so. While it is possible to have functions where this is false under relatively weak conditions partial derivatives commute, and this can be assumed. See this wiki page
 
11:34 AM
I really really wish I had more time on this assignment so I could appreciate it more
But he will give me a new one the day this is in, and then I will repeat my endless struggle to keep up
I am glad my proof isn't broken
 
The best thing to do is bring this issue up in class
 
@DavidWheeler I will. I have the statistics on my study per class, and this is absurd
 
Because it's an interesting one, and deserves some attention
 
@DavidWheeler Oh I thought you meant my not keeping up haha
@DavidWheeler But yes, I will bring it up for sure
We only have 10 students, so it is very informal in that regard
 
Lol, make your prof earn their keep
 
11:37 AM
@DavidWheeler He is really good haha, most of the questions on here were things he was asked in class though
He says "Oh that's an interesting question, that is going on your assignment"
 
Put generally, your question is actually: "can we have Jordan blocks of different sizes with the same eigenvalue"?
(in the same JNF)
You can even say, another student thinks not, but I disagree.
 
I won't say that since he is in the room and the professor will pull me up
and I have anxiety with that sort of thing, but I will ask him before class lol
 
Try to come up with some 4x4 examples, if you have time.
If you want to be diplomatic, you can say, I've heard conflicting answers-what is the truth?
 
That's probably better
 
That doesn't give away who said what.
 
11:42 AM
So does it stand that $u \in C^{2,2}$ ?? @quid
 
But yes, there is an intimate connection between the size of the Jordan block, and the power of $\lambda I - A$ we have to take the kernel of.
 
The generalised eigenvectors you called them
 
yes, we know some finite power will do, because $\lambda I_k - J_k$ will be nilpotent.
Ideally, we want a direct sum decomposition into eigenspaces (diagonalizable)
Barring that, we want a direct sum decomposition into something that "respects" the eigenspaces (hence "blocks").
 
ahhhhhhhhh yes that makes sense
 
@MaryStar yes I would say that this is usually understood that one looks for such solutions u.
 
11:49 AM
the "direct sum" part is purely algebraic, the "block" part is purely "matrix stuff"
 
@DavidWheeler So linear operators <-----> Matrices
The two 'languages' of my introduction to the course
 
Two languages for the same concepts
 
Are you my lecturer in disguise xD
 
Sometimes, the "abstract" form is cleaner, but sometimes the "concrete" form is easier to understand.
If you can learn to move fluidly between them, you have a deeper understanding of the truth behind the ideas.
 
I really like that
 
11:52 AM
Good morning, fellas
 
I think this course is amazing, just too time consuming. But maybe it needs to be so we learn everything(or are exposed to everything) within the 13 week teaching period
 
Well, linear algebra covers a LOT of ground-honestly you could fill a 2 year course with it.
 
Do I have to write at the beginning "We assume that $u \in C^{2,2}$. ?? Or do I not have to ?? @quid
 
You start out with things like x + y = 3, y - x = 1, and you wind up doing orthogonal bases in Hilbert spaces
 
@MaryStar I am sorry I do not have a definite answer to that question. But I assume you have some similar material to compare to.
 
12:00 PM
@MaryStar It couldn't hurt. That way the assumptions you are using are out in the open.
 
@DavidWheeler Well, we don't know the context do we?
 
@ABeautifulMind I presume a study of PDE's
 
Can we say: "Because $u \in C^{2,2}$ we have that $u_{xt}=u_{tx}$." ?? @quid @DavidWheeler
 
@MaryStar Yes. You know, you should think for yourself before asking. =)
 
If the function is twice continuously differentiable, the partials commute.
This is known as Schwarz' Theorem, IIRC
 
12:03 PM
@MaryStar Writing math is just writing English. Just express your thoughts. There is nothing to ask.
 
@MaryStar yes this true, I agree with @DavidWheeler This is mentioned on the site I linked too.
 
Ok... Thank you!!! :-)
 
@MaryStar I see you asking many questions and I would like to help you. Maybe you need to learn how to learn math to make progress.
 
@DavidWheeler I remember it being called Clairaut's Theorem?
 
Here is one more thing you can do-you can make the assumption, and see if you obtain a solution. Then check that your solution satisfies the problem. In that case, you can verify that $u \in C^{2,2}$. This is often done in "real life" applications.
 
12:07 PM
Are we talking about linear algebra or multivariable calculus?
 
@teadawg1337 Could be, some theorems have more than one name (independent discoveries and/or proofs)
 
Yeah, like Wheeler Loy theorem.
 
@ABeautifulMind I'm gonna hold you to that, Jasper :P
 
Hi @ABeautifulMind
 
@teadawg1337 Yes.
 
12:09 PM
@DavidWheeler To which one?
.....
 
I am trying to resolve some very difficult OCD themes next month. I need courage to do all the crazy things next month. Wish me good luck.
 
What beautiful @ᴇʏᴇs you have
 
@ᴇʏᴇs Dear Bart.
 
@teadawg1337 To one, or the other, or both.
 
12:10 PM
I wasn't expecting an exercise in mathematical logic this early in the morning, @DavidWheeler
 
I just need a way to say that therefore all the Jordan blocks are of size $1\times 1$
Oh wait I think I found a problem in my logic
 
@Committingtoachallenge I would argue this way, by (complete) induction on the size of the matrix-reduce it to JNF. If $A^n = I$, then $J^n = I$. Now we can write: $J = \begin{bmatrix}J_1&0\\0&J_2\end{bmatrix}$. Prove $J^n = \begin{bmatrix}J_1^n&0\\0&J_2^n\end{bmatrix}$
 
If $\mathscr{P}$ is a prime ideal and $\mathscr{A, B} $ usual ideals, then is it true that $(\mathscr{PA}) \cap \mathscr{B}=\mathscr{P}(\mathscr{A} \cap \mathscr{B})$?
And, in general, does anyone know of a ring where the ideals are particularly pathological, and not too hard to manipulate?
It would be really nice to have a counter-examples ring on hand.
 
12:35 PM
@DavidWheeler Done :)
 
The point of using induction is to avoid even discussing the super-diagonal, for 1x1 matrices, these are all we have, and if it's true (that the Jordan blocks are all diagonal) for any k x k matrix, where k < m, breaking the JNF into two blocks, gives us two square matrices of size less than m, so we can apply our induction hypothesis.
 
12:52 PM
@DavidW I know I didn't use induction on that(but I did fix induction for what I originally thought you were referring to), but I got rid of my $1\times 1$ argument, since the roots must be distinct. Can you see if it works please? meta.math.stackexchange.com/a/4726/142198
 
I am trying to understand correspondance between binary trees with n+1 leaves and dyck word od length 2n from this section of the wikipedia page of binary trees. Can somebody explain the last paragraph to me?
 
Hi @ABeautifulMind
 
@ᴇʏᴇs Hi.
 
@Committingtoachallenge Lol, well you used induction, but we don't need it to prove $A^n = PJ^nP^{-1}$. But that part's OK, and we can use it to show that $J^n = I$.
 
Yeah that's what I used it for haha, I thought you were saying my similarity argument was non-rigorous LOL
 
1:04 PM
The thing is, we want to show that $J^n = I \implies J$ diagonal.
Now the induction I was thinking of is the size of $A$ (so we can do it for every $n$, for all sizes).
 
Was my proof valid (although long btw)
@DavidWheeler Where does $m$ come in?
 
IN general, it is NOT true, that $A^n = I$ means $A$ is diagonal, consider $A = \begin{bmatrix}0&-1\\1&0\end{bmatrix}$ which satisfies $A^4 = I$
 
Oh woops
I meant diagonalisable
 
Hi. I wonder if someone could help me check my answer for an exercise: Assume that 28 % of voters favored party A at some point. A later opinion poll gave a result of 30 % of voters favoring part A. What is the minimum sample size allowing us to discard the null hypothesis of no change in voter preference?
Treating the poll as a binomial test approximated by the normal distribution, I arrived at n>= 1373.
 
@DavidW Better ending line: "and hence $J$ is diagonal. Since $A=P^{-1}JP$ where $J$ is diagonal, $PAP^{-1} = J$ hence $A$ is diagonalizable."
 
1:12 PM
But we're not trying to prove it for any kind of matrix, just matrices in JNF.
 
(Assuming a significance level of 5%)
 
The thing is, you haven't defined what you mean by a direct sum of matrices. Direct sums apply to subspaces (algebraic objects).
 
I had read that the direct sum of two jordan blocks looked like $\b J_1&0\\0&J_2 \e$
 
@robjohn The pinned message is still not fixed. [ in blue but ] in black. =)
 
That is a "block"decomposition, which CORRESPONDS to a direct sum decomposition.
 
1:15 PM
It takes a genius to see the above, lol.
Am I the only person to notice it?
 
@ABeautifulMind I don't see anything
 
@ᴇʏᴇs Look at [instructions] on the star wall.
 
@ABeautifulMind What is a star wall
 
@ᴇʏᴇs The wall of starred lines on the right.--------------------------------------------------->
 
@ABeautifulMind Oh I don't get that on my phone
 
1:16 PM
7 Chat guidelines | $\LaTeX$ in chat | MSE chat dwellers: pin your location (just for fun) [instructions] - mar 10 at 19:30 by Committing to a challenge
 
The point is, both $J_1,J_2$ are square matrices of some size LESS than the overall matrix.
and they are both in JNF.
So an induction hypothesis allows us to assume that $J_1^n = I$ and $J_2^n = I$ (for the appropriately sized identity matrices).
 
Ahhhh true true,
 
And by the same induction hypothesis, we are assuming that forces $J_1,J_2$ diagonal.
 
1
A: What initial value do I have to take at the beginning?

doraemonpaulFollow the method in http://en.wikipedia.org/wiki/Method_of_characteristics#Example: $\dfrac{dx}{dt}=1$ , letting $x(0)=0$ , we have $x=t$ $\dfrac{dy}{dt}=x+y=t+y$ , we have $y=y_0e^t-t-1=y_0e^x-x-1$ $\dfrac{du}{dt}=0$ , letting $u(0)=F(y_0)$ , we have $u(x,y)=F(y_0)=F((x+y+1)e^{-x})$ $u(x,1-...

Could we pick $(x(0),y(0))=(x_0,1-x_0)$ ?
 
Hi @Ramanewbie
 
1:25 PM
1
Q: What initial value do I have to take at the beginning?

evindaIn my lecture notes there is the following example on which we have applied the method of characteristics: $$u_t+2xu_x=x+u, x \in \mathbb{R}, t>0 \\ u(x,0)=1+x^2, x \in \mathbb{R}$$ $$$$ $$(x(0), t(0))=(x_0,0)$$ We will find a curve $(x(s), t(s)), s \in \mathbb{R}$ such that $\sigma '(s)=...

 
The other line of reasoning we were following (which was completely different) is that $x^n - 1$ has $n$ distinct roots. So no eigenvalue can be taken more than once.
 
@ABeautifulMind I'm not sure if there's a way to fix that, and here's why: the way to make a link is [text](url), so the extra square brackets around the text can't be interpreted correctly. The system interprets only the text between the first left and right brackets, that's why the right bracket is black on the pinned message you're referring to
 
it looks silly
 
I look silly too
 
i am silly
 
1:31 PM
@skullpatrol do you read every chat you're in
 
hi @ᴇʏᴇs ! what's up
 
most of them
 
@DavidWheeler Yeah on reflection, that argument is 1/10th a long probably
 
Here is something to test to see if you understand. The matrix $\begin{bmatrix}-\frac{1}{2}&-\frac{\sqrt{3}}{2}\\ \frac{\sqrt{3}}{2}&-\frac{1}{2}\end{bmatrix}$ satisfies $A^3 = I$. Diagonalize it.
 
Hey @DanielFischer!!!
Could you maybe take a look at this? http://math.stackexchange.com/questions/1190644/what-initial-value-do-i-have-to-take-at-the-beginning/1190678#1190678
Could we pick $(x(0),y(0))=(x_0,1-x_0)$ ?
 
1:39 PM
@teadawg1337 I see. @robjohn see teadawg's linked message, maybe he is right, I don't know.
 
1:57 PM
@evinda Think so. But I have fortunately forgotten almost all things related to PDEs, so I make no guarantees.
 
@DanielFischer I think so too :) Thanks for your answer!!!
 
@ABeautifulMind To fix it, someone would either unpin Committing to a Challenge's comment and pin a new one, or go back in the transcript and fix the error there. The link still works, so why bother?
 

« first day (1684 days earlier)      last day (3355 days later) »